Why “If Lz has a well-defined value, then Lx and Ly do not”?

In summary, Griffiths's book states that there are not three components to the angular momentum vector, a particle cannot have a well-defined angular momentum and it is impossible for two angular momentum operators not to commute.
  • #1
hilily
1
0
On Griffiths's book page 166 first paragraph he said "
It's not merely that you don't know all three components of L; there simply aren't three components—a particle just cannot have a determinate angular momentum vector, any more than it can simultaneously have a determinate position and momentum. If Lz has a well- defined value, then Lx and Ly do not."
 
Physics news on Phys.org
  • #2
This is because angular momentum operators in different directions do not commute, so it is impossible for there to be any states that are eigenstates of more than one of them. The example you quote is a state which is an eigenstate of ##L_z##, but therefore cannot be an eigenstate of any other angular momentum operator, including ##L_x## and ##L_y##.
 
  • Like
Likes hilily
  • #3
There's a single exception to this, the case where the squared angular momentum ##L^2## is zero and all of ##L_x ,L_y ,L_z## are zero as well.
 
  • Like
Likes hilily
  • #4
PeterDonis said:
This is because angular momentum operators in different directions do not commute, so it is impossible for there to be any states that are eigenstates of more than one of them. The example you quote is a state which is an eigenstate of ##L_z##, but therefore cannot be an eigenstate of any other angular momentum operator, including ##L_x## and ##L_y##.

In general, if two operators do not commute then there cannot be a complete set of common eigenstates. But, there can be some states that are common eigenstates of the non-commuting operators.
 
  • Like
Likes hilily and hilbert2
  • #5
If I have two noncommuting ##2\times 2## matrices ##\mathbf{A}## and ##\mathbf{B}##, and two diagonal ##n\times n## matrices ##\mathbf{P}## and ##\mathbf{Q}##, I can form the block matrices

##\begin{bmatrix}\mathbf{A} & 0 \\ 0 & \mathbf{P}\end{bmatrix}##

and

##\begin{bmatrix}\mathbf{B} & 0 \\ 0 & \mathbf{Q}\end{bmatrix}##,

which have an arbitrary number ##n## of linearly independent common eigenstates, but a couple of non-common ones. So actually it's possible that the non-common eigenstates are a minority, even though this doesn't happen often unless you deliberately construct this kind of a case.
 
  • Like
Likes hilily and PeroK
  • #6
hilily said:
On Griffiths's book page 166 first paragraph he said "
It's not merely that you don't know all three components of L; there simply aren't three components—a particle just cannot have a determinate angular momentum vector, any more than it can simultaneously have a determinate position and momentum. If Lz has a well- defined value, then Lx and Ly do not."
This is very misleading since a quantum system always has all the observables you can define for it, i.e., no matter in which state the system is defined you can always measure any of its observables. Of course, it depends on the state the system is prepared in, which observables take determined values. Griffiths's QT book seems to be quite sloppy in its formulations!

The other postings in this thread are of course correct concerning the question, when an observable takes a determined value (neglecting the possibility of degeneracy).
 
  • Like
Likes hilily
  • #7
hilily said:
On Griffiths's book page 166 first paragraph he said "
It's not merely that you don't know all three components of L; there simply aren't three components—a particle just cannot have a determinate angular momentum vector, any more than it can simultaneously have a determinate position and momentum. If Lz has a well- defined value, then Lx and Ly do not."
Since you're studying quantum mechanics there, you need to take the basic laws and apply those to your case. I can give you two suggestions, you have to work this out yourself.
Use these relations, and the answer will be yours:
\begin{align}
&\left[x_i,p_j\right]=i\hbar\delta_{ij}\\
&L_i=\epsilon_{ijk}x_jp_k
\end{align}
 

What does it mean for Lz to have a well-defined value?

Having a well-defined value means that Lz, or the angular momentum in the z-direction, can be measured with certainty in a given quantum system. This implies that the system has a fixed amount of angular momentum in the z-direction and that any measurement of Lz will yield the same result.

Why does Lx and Ly not have a well-defined value if Lz does?

This is a consequence of the Heisenberg uncertainty principle, which states that it is impossible to know both the position and momentum of a particle with absolute certainty. In this case, Lx and Ly represent the angular momentum in the x and y directions, respectively, and their values cannot be precisely determined if Lz is known.

How does this relate to quantum mechanics?

This statement is a fundamental principle in quantum mechanics, which is the branch of physics that studies the behavior of particles at the atomic and subatomic level. It highlights the probabilistic nature of quantum systems and the limitations of our ability to measure certain properties with precision.

What is the significance of this statement in relation to the physical world?

It is significant because it challenges our classical understanding of the world, where properties such as position and momentum are considered to have well-defined values. This statement shows that at the quantum level, these properties are inherently uncertain and can only be described in terms of probabilities.

How is this statement applied in practical applications?

This statement is applied in various fields such as quantum computing and quantum cryptography. It also has implications in technologies such as MRI machines and atomic clocks, where the principles of quantum mechanics are utilized to make precise measurements.

Similar threads

  • Quantum Physics
Replies
12
Views
1K
  • Quantum Physics
Replies
18
Views
2K
  • Advanced Physics Homework Help
Replies
12
Views
11K
  • Quantum Physics
Replies
1
Views
1K
  • Introductory Physics Homework Help
Replies
19
Views
2K
  • Advanced Physics Homework Help
Replies
12
Views
25K
Replies
1
Views
3K
Replies
18
Views
1K
Replies
86
Views
4K
Replies
11
Views
2K
Back
Top